Questions from Peskin & Schroeder 5.5 about Compton Scattering

Click For Summary
SUMMARY

The discussion focuses on the Compton scattering calculation as presented in Peskin & Schroeder (P&S), specifically addressing the amplitude evaluation and the implications of photon helicity. The amplitude is defined as $$ i\mathcal M = -ie^2 \epsilon_\mu(k)\epsilon^*_\nu(k^\prime) u_R^\dagger(p^\prime) \sigma^\mu \frac{\overline \sigma \cdot (p-k^\prime)}{-(\omega^2\chi^2+m^2)}\sigma^\nu u_R(p)$$. The participant questions the necessity of the final photon being right-handed for the amplitude to be non-zero and seeks clarification on the conservation of spin and helicity during the scattering process. Additionally, they inquire about the origin of the minus sign in the expression for the matrix element in equation (5.32).

PREREQUISITES
  • Understanding of quantum field theory concepts, particularly Compton scattering.
  • Familiarity with Peskin & Schroeder's "An Introduction to Quantum Field Theory".
  • Knowledge of spinors and their representations in particle physics.
  • Proficiency in manipulating Dirac matrices and gamma matrices in calculations.
NEXT STEPS
  • Review the derivation of the Compton scattering amplitude in Peskin & Schroeder, focusing on helicity states.
  • Study the implications of helicity conservation in particle interactions.
  • Examine the role of spinors in quantum field theory, particularly in scattering processes.
  • Investigate the derivation of matrix elements in quantum electrodynamics, specifically in relation to signs in expressions.
USEFUL FOR

Physics students, researchers in quantum field theory, and anyone studying particle interactions, particularly those interested in Compton scattering and related calculations.

Wan
Messages
3
Reaction score
0
Hi! Just a couple questions on the Compton scattering calculation in P&S. I feel like I'm missing something very simple here but can't quite figure out what it is. On page 166, the amplitude to be evaluated is
$$ i\mathcal M = -ie^2 \epsilon_\mu(k)\epsilon^*_\nu(k^\prime) u_R^\dagger(p^\prime) \sigma^\mu \frac{\overline \sigma \cdot (p-k^\prime)}{-(\omega^2\chi^2+m^2)}\sigma^\nu u_R(p),$$ where ## u_R(p) = \sqrt{2E}(0, 1)^T## and ##u_R(p^\prime) = \sqrt{2E}(1, 0)^T.## P&S argue that the final photon should be right handed otherwise the amplitude vanishes but I don't quite get that. I presumed that if the final photon is right handed then ##\epsilon^\nu = 1/\sqrt{2}(0,1,i,0)^T##, so ##\epsilon_\nu = 1/\sqrt{2}(0,-1,-i,0)^T## and ##\epsilon_\nu^* = (0, -1, i, 0)^T##, then we would have
$$
\sigma^\nu \epsilon_\nu^* = \frac{1}{\sqrt{2}} \bigg[\begin{pmatrix}
0 & 1\\
1 & 0\end{pmatrix} (-1) + \begin{pmatrix} 0 & -i \\ i & 0 \end{pmatrix} i\bigg] = \begin{pmatrix} 0 & 0 \\ -\sqrt{2} & 0 \end{pmatrix},
$$
But then ##\sigma^\nu \epsilon_\nu^*u_R(p) = \begin{pmatrix} 0 & 0 \\ -\sqrt{2} & 0 \end{pmatrix} \begin{pmatrix} 0 \\ 1 \end{pmatrix} = 0##. What did I do wrong?

Also with Figure 5.6 on the next page, I have pretty much the same question as this guy in StackExchange https://physics.stackexchange.com/questions/390121/spin-vs-helicity-conservation, unfortunately no one gave him an answer yet. My understanding is that, before the collision, we have spin/helicity of the photon = 1, spin of electron -1/2; and after the collision we have helicity of the photon = -1, and spin of electron 1/2, therefore one unit of spin is exchanged, is that correct? Any help is appreciated. Thanks!
 
Physics news on Phys.org
Welp, I wonder why there are no replies. I'll just use this opportunity to ask one more question, anyone knows how the minus sign in the second expression in (5.32) come about? For ## M(e_R^-e_L^+ \rightarrow \mu_L^-\mu_R^+) ## I have it equals to ## \overline v(p^\prime) \gamma^\mu u(p) \cdot \overline u(k) \gamma_\mu v(k^\prime) ## where ## \overline v(p^\prime) \gamma^\mu u(p) = -2E(0,1,i,0) ## (equation (5.29) which describes ##e_R^- e_L^+##) and ## \overline u(k) \gamma_\mu v(k^\prime) = -2E(0,-\cos\theta, -i, \sin\theta) ## (the equation above (5.32) describing ## \mu_L^- \mu_R^+ ##. When I dot them together I got ## M = e^2 (1-\cos\theta)##, without the minus sign. Same thing happens to ## M(e_L^-e_R^+ \rightarrow \mu_R^- \mu_L^+) ##. Where does the minus come from? Many thanks.
 

Similar threads

  • · Replies 1 ·
Replies
1
Views
3K
  • · Replies 2 ·
Replies
2
Views
2K
  • · Replies 8 ·
Replies
8
Views
3K
  • · Replies 3 ·
Replies
3
Views
2K
  • · Replies 15 ·
Replies
15
Views
3K
  • · Replies 7 ·
Replies
7
Views
3K
Replies
4
Views
3K
  • · Replies 4 ·
Replies
4
Views
2K
  • · Replies 3 ·
Replies
3
Views
3K
  • · Replies 12 ·
Replies
12
Views
4K